Quantcast
  • Register
PhysicsOverflow is a next-generation academic platform for physicists and astronomers, including a community peer review system and a postgraduate-level discussion forum analogous to MathOverflow.

Welcome to PhysicsOverflow! PhysicsOverflow is an open platform for community peer review and graduate-level Physics discussion.

Please help promote PhysicsOverflow ads elsewhere if you like it.

News

PO is now at the Physics Department of Bielefeld University!

New printer friendly PO pages!

Migration to Bielefeld University was successful!

Please vote for this year's PhysicsOverflow ads!

Please do help out in categorising submissions. Submit a paper to PhysicsOverflow!

... see more

Tools for paper authors

Submit paper
Claim Paper Authorship

Tools for SE users

Search User
Reclaim SE Account
Request Account Merger
Nativise imported posts
Claim post (deleted users)
Import SE post

Users whose questions have been imported from Physics Stack Exchange, Theoretical Physics Stack Exchange, or any other Stack Exchange site are kindly requested to reclaim their account and not to register as a new user.

Public \(\beta\) tools

Report a bug with a feature
Request a new functionality
404 page design
Send feedback

Attributions

(propose a free ad)

Site Statistics

205 submissions , 163 unreviewed
5,047 questions , 2,200 unanswered
5,345 answers , 22,709 comments
1,470 users with positive rep
816 active unimported users
More ...

  Computing box diagrams with non-vanishing external momenta

+ 2 like - 0 dislike
753 views

I'm trying to explicitly compute the following box diagram in the Feynman-t'Hooft gauge: enter image description here

If I neglect the impulsion of the $s$ quark, then the final amplitude is given by

$$\mathcal{A} \propto [\bar{s}(0) \gamma_\alpha \gamma_\delta \gamma_\mu P_L b(p_b)] \cdot [\bar{u}(p_1) \gamma^\mu \gamma^\Delta \gamma^\alpha P_L v(p_2)]\cdot I^\delta_\Delta,$$ where

$$ I^{\delta\Delta}=\int \frac{\mathrm{d}^4k}{(2\pi)^4} \frac{k^\delta(k-p_2)^\Delta}{[k^2-m_t^2][(k-p_b)^2-m_W^2][k^2-m_W^2][(k-p_2)^2-m_W^2]}.$$

If I consider vanishing external momenta $p_b$ and $p_2$, then it is easy to compute this integral and express the final amplitude in terms of $m_t$ and $m_W$. However, when I carry out this computation in the general case, I don't know how to simplify the final amplitude. More precisely, I find some expressions like

$$\mathcal{A}\propto [\bar{s}(0) \gamma_\mu p_2^\mu P_L b(p_b)] \cdot [\bar{u}(p_1) P_L v(p_2)]+\dots $$ and I don't know how to get rid of the impulsion $p_2$ in the left part of the r.h.s of the equation. I can't use Dirac's equation, because $p_2$ is in the "wrong" current and the conservation of 4-momenta is completely useless, because this would just replace $p_2$ by $p_1$.

Could you give some hints about how to simplify this expression? Some references about the effect of external momenta in box diagrams might also be helpful.

This post imported from StackExchange Physics at 2014-10-01 22:37 (UTC), posted by SE-user Melquíades
asked Sep 26, 2014 in Theoretical Physics by Melquíades (40 points) [ no revision ]
retagged Oct 1, 2014
Maybe this ref could be useful.

This post imported from StackExchange Physics at 2014-10-01 22:37 (UTC), posted by SE-user Trimok
@Trimok Thank you for the reference! It seems that they have neglected the problematic terms when they assumed that $p_1$ and $p_4$ are small (eq. 5a).

This post imported from StackExchange Physics at 2014-10-01 22:37 (UTC), posted by SE-user Melquíades

Your answer

Please use answers only to (at least partly) answer questions. To comment, discuss, or ask for clarification, leave a comment instead.
To mask links under text, please type your text, highlight it, and click the "link" button. You can then enter your link URL.
Please consult the FAQ for as to how to format your post.
This is the answer box; if you want to write a comment instead, please use the 'add comment' button.
Live preview (may slow down editor)   Preview
Your name to display (optional):
Privacy: Your email address will only be used for sending these notifications.
Anti-spam verification:
If you are a human please identify the position of the character covered by the symbol $\varnothing$ in the following word:
p$\hbar$ysics$\varnothing$verflow
Then drag the red bullet below over the corresponding character of our banner. When you drop it there, the bullet changes to green (on slow internet connections after a few seconds).
Please complete the anti-spam verification




user contributions licensed under cc by-sa 3.0 with attribution required

Your rights
...